OG Quant Review #82

This topic has expert replies
Junior | Next Rank: 30 Posts
Posts: 26
Joined: Mon Oct 27, 2008 1:01 pm
Thanked: 1 times

OG Quant Review #82

by 480ocean » Sun Feb 15, 2009 1:04 pm
If m and n are consecutive positive integers, is m greater than n?
1. m-1 and n+1 are consecutive positive integers.
2. m is an even integer.

Answer is A

Thanks for your help!

Legendary Member
Posts: 2467
Joined: Thu Aug 28, 2008 6:14 pm
Thanked: 331 times
Followed by:11 members

by cramya » Sun Feb 15, 2009 1:36 pm
Take easier of the the two

Stmt II

No info about n .INSUFF

Stmt I

Lets assume that:
The order can be mn(m<n) or nm((m>n)

If its mn then m-1 and n+1 can never be consecutive. Therefore it has to be nm where m-1 (i.e. n) and n+1 (i.e m) are again conseutive satisfying Stmt I

SUFF

Hence A

Master | Next Rank: 500 Posts
Posts: 344
Joined: Sun Sep 28, 2008 11:00 am
Location: USA
Thanked: 6 times
Followed by:1 members

by Bidisha800 » Sun Feb 15, 2009 2:03 pm
m-1 and n+1 are consecutive ,

therefore, either (m-1) + 1= (n+1) OR m-1 = (n+1) +1


if (m-1) + 1= (n+1)

m= n+1

therefore, m>n

if m-1 = (n+1) +1

m = n +3

m>n

(A) suff

(B) is insuff
Drill baby drill !

GMATPowerPrep Test1= 740
GMATPowerPrep Test2= 760
Kaplan Diagnostic Test= 700
Kaplan Test1=600
Kalplan Test2=670
Kalplan Test3=570

Junior | Next Rank: 30 Posts
Posts: 26
Joined: Mon Oct 27, 2008 1:01 pm
Thanked: 1 times

by 480ocean » Mon Feb 16, 2009 2:08 pm
Got it! Thank you for your help!